In $ {\ bf R} ^ {n \ times p} $ haben wir also das innere Frobenius-Produkt gegeben durch $$ \ langle A, B \ rangle = \ text {tr} (A ^ TB) $$

, was als das euklidische innere Produkt auf $ {\ bf R} ^ {np interpretiert werden kann } $. Mein Verständnis ist, dass alle inneren Produkte auf $ {\ bf R} ^ {np} $ als $$ a ^ TPb $$ für $ P $ positiv-definit geschrieben werden können. Das Beste, was ich tun könnte, um das innere Produkt von Frobenius auf $ {\ bf R} ^ {n \ times p} $ zu erweitern, ist etwas in der Form $$ \ langle A, B \ rangle = \ sum_ {i = 1} ^ N \ text {tr} ((X_iAY_i) ^ T (X_iBY_i)) $$ für $ X_i \ in {\ bf R} ^ {m_i \ times n} $ und $ Y_i \ in {\ bf R} ^ {p \ times q_i} $ all full rank. Ich würde jedoch gerne wissen, ob dies alle inneren Produkte auf $ {\ bf R} ^ {np} $ abdeckt oder ob es aufgrund von Redundanzen komplexer als nötig ist.

Ich kann das finden entsprechende $ P $ -Matrix für ein bestimmtes Matrix-Innenprodukt, indem die Standardbasis für $ {\ bf R} ^ {n \ times p} $ verwendet und die Matrix

\ begin {bmatrix} \ langle E_1 gebildet wird , E_1 \ rangle & \ langle E_1, E_2 \ rangle & \ dots & \ langle E_1, E_ {np} \ rangle \\ \ langle E_2, E_1 \ rangle & \ langle E_2, E_2 \ rangle & & \ vdots \\ \ vdots & & \ ddots \\ \ langle E_ {np }, E_1 \ rangle & \ dots & \ dots & \ langle E_ {np }, E_ {np} \ rangle \ end {bmatrix}

, aber ich weiß nicht, ob die allgemeine Form für ein Matrix-Innenprodukt, die ich oben angegeben habe, alle positiv-definierten Matrizen abdeckt $ P $.

Update:

neuere Version dieser Frage am MathOverflow: https://mathoverflow.net/questions/229675/extending-the-trace-inner-product-to-all-matrix-real-inner-products

Kommentare

  • Willkommen bei SciComp.SE! Dies ist eine interessante Frage, scheint jedoch für math.stackexchange.com viel geeigneter zu sein. (Sofern nicht ‚ eine Verbindung zu einem rechnerwissenschaftlichen Problem besteht, fehlt mir ‚. In diesem Fall ‚ wäre großartig, wenn Sie das hinzufügen könnten.)
  • @ChristianClason, ‚ bezieht sich seit Riemannian auf die Optimierung von Matrixverteilern mit Riemannschen Metriken Metriken sind innere Produkte im Tangentenraum. ‚ ist mit ziemlicher Sicherheit zu weit fortgeschritten für Math.SE, der einzige andere geeignete Ort wäre MathOverflow. Ich habe vielleicht tatsächlich eine Lösung gefunden, die ich als Antwort poste, sobald ich die unordentliche Arbeit erledigt habe, zu beweisen, dass es sich um eine Lösung handelt, aber wenn Sie ‚ migrieren möchten Dies zu MathOverflow Ich ‚ bin damit einverstanden. Ich ‚ füge den Optimierungskontext hinzu, wenn ich eine Chance bekomme.
  • Die Matrix $ P $ muss auch symmetrisch sein, nicht nur positiv definitiv.
  • @WolfgangBangerth, positiv-definit bedeutet symmetrisch.
  • Nicht für alle Autoren impliziert positive Definitivität Symmetrie.

Antwort

Sie können ein inneres Produkt als eine Operation sehen $ f (a, b) = \ left < a, b \ right > $, dh es ist eine bilineare Funktion, die (i) eine nicht negative Zahl zurückgibt, (ii) die Beziehung $ f (a, b) = f (b, a) $.

Für die Vektoren $ a, b \ in \ mathbb R ^ n $ können alle bilinearen Funktionen, die diese Eigenschaften erfüllen, als $$ f (a, b) = \ sum_ {i, j = 1 geschrieben werden } ^ n a_i P_ {ij} b_j $$ wobei $ P $ symmetrisch und positiv definit ist. Für die Matrizen $ a, b \ in \ mathbb R ^ {n \ times p} $ können alle diese Funktionen geschrieben werden als $$ f (a, b) = \ sum_ {i, k = 1} ^ n \ sum_ { j, l = 1} ^ p a_ {ij} P_ {ijkl} b_ {kl} $$ wobei jetzt $ P $ ein Tensor von Rang 4 ist, der in dem Sinne symmetrisch ist, dass $ P_ {ijkl} = P_ {klij} $ und positiv definitiv in dem Sinne, dass $ f (a, a) > 0 $ für alle $ a \ neq 0 $.

Ihre Frage läuft darauf hinaus ob jedes $ P $, das solche Bedingungen erfüllt, eine Form geschrieben werden kann, die sich aus den Vektoren $ X_i, Y_i $ ergibt. Ich glaube, die Antwort darauf ist nein. Dies ist einfach so, weil das (der Einfachheit halber angenommene $ n = p $) symmetrische $ P $ (asymptotisch) $ n ^ 4/2 $ Freiheitsgrade hat, während die $ n $ Vektoren $ X_i, Y_i $ nur $ 2n haben ^ 2 $ Freiheitsgrade. Mit anderen Worten, ich glaube nicht, dass Ihr Ansatz für ausreichend große $ n $ ausreichend viele Freiheitsgrade hat.

Kommentare

  • I. Ich glaube tatsächlich, dass die Antwort Ja lautet. Ich werde diese Frage zum mathematischen Überlauf mit meinen aktualisierten Ergebnissen erneut veröffentlichen.
  • Ja, Ihr Argument, dass die Anzahl der Parameter zunimmt vierteljährlich im inneren Produktraum des Vektors, während nur quadratisch im inneren Produktraum der Matrix zwingend ist. Da der Raum jedoch letztendlich endlich ist, sollten wir dies überwinden können, indem wir $ N $ angemessen erhöhen.
  • Ich entschuldige mich, dass ich eine neuere Version dieser Frage auf MathOverflow gepostet habe. ‚ ist jedoch ausreichend aktualisiert. Ich hielt dies für angemessen. Hier ist der Link, falls Sie dies wünschen um Ihre Antwort dort zu übertragen oder Ihre Antwort basierend auf der neueren Version zu aktualisieren. mathoverflow.net/questions/229675/…
  • @Thoth Beachten Sie, dass @ ChristianClason empfohlen hat Sie können Ihre Frage auf math.stackexchange.com und nicht auf mathoverflow.net posten. Dies sind zwei verschiedene Websites mit unterschiedlichen Zwecken und Zielgruppen.
  • @FedericoPoloni Ja, ich weiß, und wenn Sie lesen, was ich geschrieben habe, sagte ich ihm, dass ich dachte, es sei zu weit fortgeschritten für Math.SE und würde es wahrscheinlich nicht bekommen eine Antwort dort.

Schreibe einen Kommentar

Deine E-Mail-Adresse wird nicht veröffentlicht. Erforderliche Felder sind mit * markiert.